Use non-breaking space (~) between 'Figure' and the figure number.
[course.git] / latex / problems / Serway_and_Jewett_8 / problem03.43.tex
index c3442ce946e7ff2821ac44fa9230877f915b97b2..f64dd635eb36ebc42dd25979cd3b419de78e7895 100644 (file)
@@ -6,7 +6,7 @@ airplane is directly above you so that the vector leading from you to
 it is $\vect{P}_0=7.60\E{3}\jhat\U{m}$.  At $t=30.0\U{s}$, the
 position vector leading from you to the airplane is
 $\vect{P}_{30}=(8.04\E{3}\ihat+7.60\E{3}\jhat)\U{m}$ as suggested in
 it is $\vect{P}_0=7.60\E{3}\jhat\U{m}$.  At $t=30.0\U{s}$, the
 position vector leading from you to the airplane is
 $\vect{P}_{30}=(8.04\E{3}\ihat+7.60\E{3}\jhat)\U{m}$ as suggested in
-Figure P3.43.  Determine the magnitude and orientation of the
+Figure~P3.43.  Determine the magnitude and orientation of the
 airplane's position vector at $t=45.0\U{s}$.
 \begin{center}
 \begin{asy}
 airplane's position vector at $t=45.0\U{s}$.
 \begin{center}
 \begin{asy}